LSAT and Law School Admissions Forum

Get expert LSAT preparation and law school admissions advice from PowerScore Test Preparation.

User avatar
 Dave Killoran
PowerScore Staff
  • PowerScore Staff
  • Posts: 5852
  • Joined: Mar 25, 2011
|
#26442
Complete Question Explanation
(The complete setup for this game can be found here: lsat/viewtopic.php?t=8639)

The correct answer choice is (E)

A program that cannot be ranked third is one that has either three or more variables ranked before it, or five or more variables ranked behind it (such as H). Applying the former criterion produces the following analysis:

  • H ..... minimum of 0 variables ranked ahead, cannot be ranked third since must be ranked first
    J ..... minimum of 1 variable ranked ahead (H), can be ranked third
    L ..... minimum of 1 variable ranked ahead (H), can be ranked third
    P ..... minimum of 3 variables ranked ahead (H, J, Q), cannot be ranked third
    Q ..... minimum of 2 variables ranked ahead (H, J), can be ranked third
    S ..... minimum of 4 variables ranked ahead (H, J, Q, L), cannot be ranked third
    V ..... minimum of 2 variables ranked ahead (H, L), can be ranked third
Accordingly, H, P, and S cannot be ranked third. Since only P appears among the answer choices, answer choice (E) is correct.

Note that this question does not require any writing. The listing above has been provided for the purpose of clarity. Since speed is a factor during the test, the number of variables ranked ahead should be visually scanned and counted.
 buyer3700
  • Posts: 5
  • Joined: May 09, 2012
|
#4412
In the answer to the setup, it says that either P or V must be last. But then in #3, it says that V could be 3rd, because only H,L are ahead of it. This is a contradiction no? This is a Global question, so the main setup should hold for this question.

And... can you explain #4 also. I do not understand how we get from our original diagram to the setup for this Local question. Once we get the setup, I understand how to get the answer, but I am having a hard time reading the sequencing setups.

:x :-? :x
 Steve Stein
PowerScore Staff
  • PowerScore Staff
  • Posts: 1153
  • Joined: Apr 11, 2011
|
#4415
Thanks for your question--that setup is actually not a contradiction. If either P or V must be last, and we find out that V is third, then that just means P has to fill in that last space.

Please let me know whether this makes sense--thanks!

~Steve
 kmaragh
  • Posts: 8
  • Joined: Jun 17, 2020
|
#76308
I still don't understand why answer choice A or B is incorrect. L or J will be second depending on which way the local rules are spelled out. So they will never be ranked third since it has to be HJQ or HLV. How would J or L be able to be ranked third if according to the rules one of them will always be second? I see why P also is the answer since it has too many variables ahead of it to be ranked third but J or L couldn't be third since one or the other will be second, therefore they also will not be ranked third. can someone please break this down for me.
 Christen Hammock
PowerScore Staff
  • PowerScore Staff
  • Posts: 61
  • Joined: May 14, 2020
|
#76325
Hi Kmaragh!

Just because J or L must be second doesn't mean that the other can't be third! We could see something like this—H/J/L or H/L/J—as our first three variables without breaking any rules.

Get the most out of your LSAT Prep Plus subscription.

Analyze and track your performance with our Testing and Analytics Package.